You are on page 1of 25

Mathematics Olympiad Coachs Seminar, Zhuhai, China 1

03/22/2004

Algebra
1. There exists a polynomial P of degree 5 with the following property: if z is a complex number such
that z 5 + 2004z = 1, then P (z 2 ) = 0. Find all such polynomials P .

2. Let N denote the set of positive integers. Find all functions f : N → N such that

f (m + n)f (m − n) = f (m2 )

for all m, n ∈ N.

Solution: Function f (n) = 1, for all n ∈ N, is the only function satisfying the conditions of
the problem.
Note that
f (1)f (2n − 1) = f (n2 ) and f (3)f (2n − 1) = f ((n + 1)2 )
for n ≥ 3. Thus
f (3) f ((n + 1)2 )
= .
f (1) f (n2 )
f (3)
Setting f (1) = k yields f (n2 ) = k n−3 f (9) for n ≥ 3. Similarly, for all h ≥ 1,

f (h + 2) f ((m + 1)2 )
=
f (h) f (m2 )

for sufficiently large m and is thus also k. Hence f (2h) = k h−1 f (2) and f (2h + 1) = k h f (1).
But
f (25) f (25) f (11)
= · ··· · = k8
f (9) f (23) f (9)
and
f (25) f (25) f (16)
= · = k2 ,
f (9) f (16) f (9)
so k = 1 and f (16) = f (9). This implies that f (2h + 1) = f (1) = f (2) = f (2j) for all j, h, so f is
constant. From the original functional equation it is then clear that f (n) = 1 for all n ∈ N.

3. Call a real-valued function f very convex if


µ ¶
f (x) + f (y) x+y
≥f + |x − y|
2 2

holds for all real numbers x and y. Prove that no very convex function exists.

First Solution: Fix n ≥ 1. For each integer i, define


µ ¶ µ ¶
i+1 i
∆i = f −f .
n n
2 Zuming Feng (zfeng@exeter.edu), Phillips Exeter Academy, Exeter 03833, USA

The given inequality with x = (i + 2)/n and y = i/n implies


¡ i+2 ¢ ¡i¢ µ ¶
f n +f n i+1 2
≥f + ,
2 n n

from which,
µ ¶ µ ¶ µ ¶ µ ¶
i+2 i+1 i+1 i 4
f −f ≥f −f + ,
n n n n n
that is, ∆i+1 ≥ ∆i + 4/n. Combining this for n consecutive values of i gives ∆i+n ≥ ∆i + 4. Summing
this inequality for i = 0 to i = n − 1 and cancelling terms yields

f (2) − f (1) ≥ f (1) − f (0) + 4n.

This cannot hold for all n ≥ 1. Hence there are no very convex functions.

Second Solution: We show by induction that the given inequality implies


µ ¶
f (x) + f (y) x+y
−f ≥ 2n |x − y|
2 2

for all nonnegative integers n. This will yield a contradiction, because for fixed x and y the right side
gets arbitrarily large, while the left side remains fixed.
We are given the base case n = 0. Now if the inequality holds for a given n, then for a, b real,

f (a) + f (a + 2b)
≥ f (a + b) + 2n+1 |b|,
2
f (a + b) + f (a + 3b) ≥ 2(f (a + 2b) + 2n+1 |b|),

and
f (a + 2b) + f (a + 4b)
≥ f (a + 3b) + 2n+1 |b|.
2
Adding these three inequalities and canceling terms yields

f (a) + f (a + 4b)
≥ f (a + 2b) + 2n+3 |b|.
2

Setting x = a, y = a + 4b, we obtain


µ ¶
f (x) + f (y) x+y
≥f + 2n+1 |x − y|,
2 2

and the induction is complete.

Third Solution: Rewrite the condition as


µ ¶ µ ¶
x+y x+y
f (x) − f ≥f − f (y) + 2|x − y|.
2 2
Mathematics Olympiad Coachs Seminar, Zhuhai, China 3

For any positive integer n,

f (1) − f (0) = f (1) − f (1/2) + f (1/2) − f (0)


≥ f (1/2) − f (0) + 2 + f (1/2) − f (0)
= 2[f (1/2) − f (0)] + 2
= 2[f (1/2) − f (1/4) + f (1/4) − f (0)] + 2
≥ 2[f (1/4) − f (0) + 1 + f (1/4) − f (0)] + 2
= 4[f (1/4) − f (0)] + 4 = · · · ≥ 2n [f (1/2n ) − f (0)] + 2n.

Similarly, f (−1) − f (0) ≥ 2n [f (−1/2n ) − f (0)] + 2n. But

f (1/2n ) + f (−1/2n ) ≥ 2f (0) + 1/2n−2 > 2f (0).

Thus, for each n, at least one of f (1/2n ) − f (0) and f (−1/2n ) − f (0) is greater than 0. It follows
that at least one of f (1) − f (0) and f (−1) − f (0) is greater than 2n for all n ≥ 1, which is impossible.
Hence there is no very convex functions.

4. Let a1 , a2 , . . . , an (n > 3) be real numbers such that

a1 + a2 + · · · + an ≥ n and a21 + a22 + · · · + a2n ≥ n2 .

Prove that max(a1 , a2 , . . . , an ) ≥ 2.

P P
Solution: Let bi = 2 − ai , and let S = bi and T = b2i . Then the given conditions are
that
(2 − a1 ) + · · · + (2 − an ) ≥ n
and
(4 − 4b1 + b21 ) + · · · + (4 − 4bn + b2n ) ≥ n2 ,
which is to say S ≤ n and T ≥ n2 − 4n + 4S.
From these inequalities, we obtain

T ≥ n2 − 4n + 4S ≥ (n − 4)S + 4S = nS.
P
On the other hand, if bi > 0 for i = 1, . . . , n, then certainly bi < bi = S ≤ n, and so

T = b21 + · · · + b2n < nb1 + · · · + nbn = nS.

Thus we cannot have bi > 0 for i = 1, . . . , n, so bi ≤ 0 for some i, and ai ≥ 2 for that i, proving the
claim.

Note: The statement is false when n ≤ 3. The example a1 = a2 = · · · = an−1 = 2, an = 2 − n


shows
P that the Pbound cannot be improved. An alternate approach is to show that if ai ≤ 2 and
ai ≥ n, then a2i ≤ n2 (with the equality case just mentioned), by noticing that replacing a pair
ai , aj with 2, ai + aj − 2 increases the sum of squares.
1
5. Let { an }n≥0 be a sequence of real numbers such that an+1 ≥ a2n + for all n ≥ 0. Prove that
√ 5
an+5 ≥ a2n−5 for all n ≥ 5.
4 Zuming Feng (zfeng@exeter.edu), Phillips Exeter Academy, Exeter 03833, USA

First Solution: (by Alison Miller) For any k, we have the following inequality:
1 1 1 1
ak+1 + ≥ a2k + + = a2k + ≥ ak ,
20 5 20 4
because (ak − 12 )2 ≥ 0. Summing up this inequalities over k = n + 1, . . . , n + 4, we have

1 1
an+5 + ≥ an+1 ≥ a2n + ,
5 5
and similarly an ≥ a2n−5 , so
an+5 ≥ a2n ≥ a4n−5 ,
implying the desired result.

6. Prove that the average of the numbers n sin n◦ (n = 2, 4, 6, . . . , 180) is cot 1◦ .

Solution: All arguments of trigonometric functions will be in degrees. We need to prove

2 sin 2 + 4 sin 4 + · · · + 178 sin 178 = 90 cot 1, (∗)

which is equivalent to

2 sin 2 · sin 1 + 2(2 sin 4 · sin 1) + · · · + 89(2 sin 178 · sin 1) = 90 cos 1.

Using the identity 2 sin a · sin b = cos(a − b) − cos(a + b), we find

2 sin 2 · sin 1 + 2(2 sin 4 · sin 1) + · · · + 89(2 sin 178 · sin 1)


= (cos 1 − cos 3) + 2(cos 3 − cos 5) + · · · + 89(cos 177 − cos 179)
= cos 1 + cos 3 + cos 5 + · · · + cos 175 + cos 177 − 89 cos 179
= cos 1 + (cos 3 + cos 177) + · · · + (cos 89 + cos 91) − 89 cos 179
= cos 1 + 89 cos 1 = 90 cos 1,

so (∗) is true.

7. Let a, b, c be real numbers in the interval (0, π2 ). Prove that

sin a sin(a − b) sin(a − c) sin b sin(b − c) sin(b − a) sin c sin(c − a) sin(c − b)


+ + ≥ 0.
sin(b + c) sin(c + a) sin(a + b)

Solution: By the Product-to-sum formulas and the Double-angle formulas, we have


1
sin(α − β) sin(α + β) = [cos 2β − cos 2α]
2
= sin2 α − sin2 β.

Hence, we obtain

sin a sin(a − b) sin(a − c) sin(a + b) sin(a + c)


= sin c(sin2 a − sin2 b)(sin2 a − sin2 c)
Mathematics Olympiad Coachs Seminar, Zhuhai, China 5

and its analogous forms. Therefore, it suffices to prove that

x(x2 − y 2 )(x2 − z 2 ) + y(y 2 − z 2 )(y 2 − x2 ) + z(z 2 − x2 )(z 2 − y 2 ) ≥ 0,

where x = sin a, y = sin b, and z = sin c (hence x, y, z > 0). Since the last inequality is symmetric
with respect to x, y, z, we may assume that x ≥ y ≥ z > 0. It suffices to prove that

x(y 2 − x2 )(z 2 − x2 ) + z(z 2 − x2 )(z 2 − y 2 ) ≥ y(z 2 − y 2 )(y 2 − x2 ),

which is evident as
x(y 2 − x2 )(z 2 − x2 ) ≥ 0
and
z(z 2 − x2 )(z 2 − y 2 ) ≥ z(y 2 − x2 )(z 2 − y 2 ) ≥ y(z 2 − y 2 )(y 2 − x2 ).

Note: The key step of the proof is an instance of Schur’s Inequality with r = 21 .

8. Let ABC be a triangle. Prove that


3A 3B 3C A−B B−C C −A
sin + sin + sin ≤ cos + cos + cos .
2 2 2 2 2 2

First Solution: Let α = A2 , β = B2 , γ = C2 . Then 0◦ < α, β, γ < 90◦ and α + β + γ = 90◦ .


By the Difference to Product formulas, we have
3A B−C
sin − cos = sin 3α − cos(β − γ)
2 2
= sin 3α − sin(α + 2γ)
= 2 cos(2α + γ) sin(α − γ)
= −2 sin(α − β) sin(α − γ).

In exactly the same way, we can show that


3B C −A
sin − cos = −2 sin(β − α) sin(β − γ)
2 2
and
3C A−B
sin− cos = −2 sin(γ − α) sin(γ − β).
2 2
Hence it suffices to prove that

sin(α − β) sin(α − γ) + sin(β − α) sin(β − γ)


+ sin(γ − α) sin(γ − β) ≥ 0.

Note that this inequality is symmetric with respect to α, β, γ, so we can assume without loss of
generality that 0◦ < α < β < γ < 90◦ . Then regrouping the terms on the left-hand side gives

sin(α − β) sin(α − γ) + sin(γ − β)[sin(γ − α) − sin(β − α)],

which is positive because the function y = sin x is increasing for 0◦ < x < 90◦ .
6 Zuming Feng (zfeng@exeter.edu), Phillips Exeter Academy, Exeter 03833, USA

Note: This proof is similar to that of the Schur’s Inequality.

Second Solution: We keep the notation of the first solution. By the Addition formulas,
we have

sin 3α = sin α cos 2α + sin 2α cos α;


cos(β − α) = sin(2α + γ) = sin 2α cos γ + sin γ cos 2α;
cos(β − γ) = sin(2γ + α) = sin 2γ cos α + sin α cos 2γ;
sin 3γ = sin γ cos 2γ + sin 2γ cos γ.

By the Difference to Product formulas, it follows that

sin 3α + sin 3γ − cos(β − α) − cos(β − γ)


= (sin α − sin γ)(cos 2α − cos 2γ)
+(cos α − cos γ)(sin 2α − sin 2γ)
= (sin α − sin γ)(cos 2α − cos 2γ)
+2(cos α − cos γ) cos(α + γ) sin(α − γ).

Note that sin x is increasing, cos x and cos(2x) are decreasing for 0 < x < 90◦ . Since 0 < α, γ, α +γ <
90◦ , each of the two products in the last addition is less than or equal to 0. Hence

sin 3α + sin 3γ − cos(β − α) − cos(β − γ) ≤ 0.

In exactly the same way, we can show that

sin 3β + sin 3α − cos(γ − β) − cos(γ − α) ≤ 0

and
sin 3γ + sin 3β − cos(α − γ) − cos(α − β) ≤ 0.

Adding the last three inequalities gives the desired result.

9. Let a, b, c be positive real numbers. Prove that

(2a + b + c)2 (2b + c + a)2 (2c + a + b)2


+ + ≤ 8.
2a2 + (b + c)2 2b2 + (c + a)2 2c2 + (a + b)2

First Solution: (Based on work by Matthew Tang and Anders Kaseorg) By multiplying a, b,
and c by a suitable factor, we reduce the problem to the case when a + b + c = 3. The desired
inequality reads
(a + 3)2 (b + 3)2 (c + 3)2
+ + ≤ 8.
2a2 + (3 − a)2 2b2 + (3 − b)2 2c2 + (3 − c)2
Set
(x + 3)2
f (x) =
2x2 + (3 − x)2
Mathematics Olympiad Coachs Seminar, Zhuhai, China 7

It suffices to prove that f (a) + f (b) + f (c) ≤ 8. Note that


x2 + 6x + 9 1 x2 + 6x + 9
f (x) = = ·
3(x2 − 2x + 3) 3 x2 − 2x + 3
µ ¶ µ ¶
1 8x + 6 1 8x + 6
= 1+ 2 = 1+
3 x − 2x + 3 3 (x − 1)2 + 2
µ ¶
1 8x + 6 1
≤ 1+ = (4x + 4).
3 2 3
Hence,
1
f (a) + f (b) + f (c) ≤ (4a + 4 + 4b + 4 + 4c + 4) = 8,
3
as desired, with equality if and only if a = b = c.

Second Solution: (By Liang Qin) Setting x = a + b, y = b + c, z = c + a gives 2a + b + c = x + z,


hence 2a = x + z − y and their analogous forms. The desired inequality becomes
2(x + z)2 2(z + y)2
+
(x + z − y)2 + 2y 2 (z + y − x)2 + 2x2
2(y + x)2
+ ≤ 8.
(y + x − z)2 + 2z 2
Because 2(s2 +t2 ) ≥ (s+t)2 for all real numbers s and t, we have 2(x+z −y)2 +2y 2 ≥ (x+z −y+y)2 =
(x + z)2 . Hence
2(x + z)2
(x + z − y)2 + 2y 2
4(x + z)2 4(x + z)2
= ≤
2(x + z − y)2 + 4y 2 (x + z)2 + 2y 2
4 4 4(x2 + z 2 )
= ≤ = .
y 2
1 + 2 · (x+z) 2
2
1 + 2 · 2(x2y+z 2 ) x2 + y 2 + z 2

It is not difficult to see that the desired result follows from summing up the above inequality and its
analogous forms.

Third Solution: (By Richard Stong) Note that

(2x + y)2 + 2(x − y)2 = 4x2 + 4xy + y 2 + 2x2 − 4xy + 2y 2


= 3(2x2 + y 2 ).

Setting x = a and y = b + c yields

(2a + b + c)2 + 2(a − b − c)2 = 3(2a2 + (b + c)2 ).

Thus, we have
(2a + b + c)2 3(2a2 + (b + c)2 ) − 2(a − b − c)2
=
2a2 + (b + c)2 2a2 + (b + c)2
2(a − b − c)2
= 3− 2 .
2a + (b + c)2
8 Zuming Feng (zfeng@exeter.edu), Phillips Exeter Academy, Exeter 03833, USA

and its analogous forms. Thus, the desired inequality is equivalent to


(a − b − c)2 (b − a − c)2 (c − a − b)2 1
2 2
+ 2 2
+ 2 2
≥ .
2a + (b + c) 2b + (c + a) 2c + (a + b) 2

Because (b + c)2 ≤ 2(b2 + c2 ), we have 2a2 + (b + c)2 ≤ 2(a2 + b2 + c2 ) and its analogous forms. It
suffices to show that
(a − b − c)2 (b − a − c)2 (c − a − b)2 1
+ + ≥ ,
2(a2 + b2 + c2 ) 2(a2 + b2 + c2 ) 2(a2 + b2 + c2 ) 2
or,
(a − b − c)2 + (b − a − c)2 + (c − a − b)2 ≥ a2 + b2 + c2 .
Multiplying this out, the left-hand side of the last inequality becomes 3(a2 + b2 + c2 ) − 2(ab + bc + ca).
Therefore the last inequality is equivalent to 2[a2 + b2 + c2 − (ab + bc + ca)] ≥ 0, which is evident
because
2[a2 + b2 + c2 − (ab + bc + ca)] = (a − b)2 + (b − c)2 + (c − a)2 .
Equalities hold if and only if (b + c)2 = 2(b2 + c2 ) and (c + a)2 = 2(c2 + a2 ), that is, a = b = c.

Fourth Solution: We first convert the inequality into


2a(a + 2b + 2c) 2b(b + 2c + 2a) 2c(c + 2a + 2b)
+ + ≤ 5.
2a2 + (b + c)2 2b2 + (c + a)2 2c2 + (a + b)2
Splitting the 5 among the three terms yields the equivalent form
X 4a2 − 12a(b + c) + 5(b + c)2
≥ 0, (1)
cyc
3[2a2 + (b + c)2 ]
P
where cyc is the cyclic sum of variables (a, b, c). The numerator of the term shown factors as
(2a − x)(2a − 5x), where x = b + c. We will show that
(2a − x)(2a − 5x) 4(2a − x)
2 2
≥− . (2)
3(2a + x ) 3(a + x)
Indeed, (2) is equivalent to

(2a − x)[(2a − 5x)(a + x) + 4(2a2 + x2 )] ≥ 0,

which reduces to
(2a − x)(10a2 − 3ax − x2 ) = (2a − x)2 (5a + x) ≥ 0,
which is evident. We proved that
4a2 − 12a(b + c) + 5(b + c)2 4(2a − b − c)
2 2
≥− ,
3[2a + (b + c) ] 3(a + b + c)
hence (1) follows. Equality holds if and only if 2a = b + c, 2b = c + a, 2c = a + b, i.e., when a = b = c.

Fifth Solution: Given a function f of n variables, we define the symmetric sum


X X
f (x1 , . . . , xn ) = f (xσ(1) , . . . , xσ(n) )
sym σ
Mathematics Olympiad Coachs Seminar, Zhuhai, China 9

where σ runs over all permutations of 1, . . . , n (for a total of n! terms). For example, if n = 3, and
we write x, y, z for x1 , x2 , x3 ,
X
x3 = 2x3 + 2y 3 + 2z 3
sym
X
x2 y = x2 y + y 2 z + z 2 x + x2 z + y 2 x + z 2 y
sym
X
xyz = 6xyz.
sym

We combine the terms in the desired inequality over a common denominator and use symmetric sum
notation to simplify the algebra. The numerator of the difference between the two sides is
X
2 4a6 + 4a5 b + a4 b2 + 5a4 bc + 5a3 b3 − 26a3 b2 c + 7a2 b2 c2 , (3)
sym

and it suffices to show the the expression in (3) is always greater or equal to 0. By the Weighted
AM-GM Inequality, we have 4a6 + b6 + c6 ≥ 6a4 bc, 3a5 b + 3a5 c + b5 a + c5 a ≥ 8a4 bc, and their
analogous forms. Adding those inequalities yields
X X X X
6a6 ≥ 6a4 bc and 8a5 b ≥ 8a4 bc.
sym sym sym sym

Consequently, we obtain
X X
4a6 + 4a5 b + 5a4 bc ≥ 13a4 bc. (4)
sym sym
Again by the AM-GM Inequality, we have a4 b2 + b4 c2 + c4 a2 ≥ 4a2 b2 c2 , a3 b3 + b3 c3 + c3 a3 ≥ 3a2 b2 c2 ,
and their analogous forms. Thus,
X X
a4 b2 + 5a3 b3 ≥ 6a2 b2 c2 ,
sym sym
or X X
a4 b2 + 5a3 b3 + 7a2 b2 c2 ≥ 13a2 b2 c2 . (5)
sym sym
Recalling Schur’s Inequality, we have

a3 + b3 + c3 + 3abc − (a2 b + b2 c + c2 a + ab2 + bc2 + ca2 )


= a(a − b)(a − c) + b(b − a)(b − c) + c(c − a)(c − b) ≥ 0,

or X
a3 − 2a2 b + abc ≥ 0.
sym
Thus X X
13a4 bc − 26a3 b2 c + 13a2 b2 c2 ≥ 13abc a3 − 2a2 b + abc ≥ 0. (6)
sym sym
Adding (4), (5), (6) yields (3).
10 Zuming Feng (zfeng@exeter.edu), Phillips Exeter Academy, Exeter 03833, USA

Note: While the last two methods seem inefficient for this problem, they hold the keys to proving
the following inequality:

(b + c − a)2 (c + a − b)2 (a + b − c)2 3


+ + ≥ ,
(b + c)2 + a2 (c + a)2 + b2 (a + b)2 + c2 5

where a, b, c are positive real numbers.

10. Let a, b, c be nonnegative real numbers. Prove that

a+b+c √ √ √ √ √ √ √
− abc ≤ max{( a − b)2 , ( b − c)2 , ( c − a)2 }.
3

First Solution: We prove the stronger inequality


√ √ √ √ √ √ √
a + b + c − 3 abc ≤ ( a − b)2 + ( b − c)2 + ( c − a)2 .
3
(1)

The conclusion is immediate if abc = 0, so we assume that a, b, c > 0. By multiplying a, b, c by a


suitable factor, we may reduce to the case abc = 1. Without loss of generality, assume that a and
b are both greater than or equal to 1, or both less than or equal to 1. The desired inequality now
becomes
√ √ √
0 ≤ a + b + c − 2 ab − 2 bc − 2 ca + 3
√ √ 1 2 2
= ( a − b)2 + − √ − √ +3
ab a b
µ ¶2 µ ¶2
√ √ 2 1 1
= ( a − b) + √ − 1 + √ − 1
a b
1 1 1
+ − − +1
ab a b
µ ¶2 µ ¶2
√ √ 2 1 1
= ( a − b) + √ − 1 + √ − 1
a b
µ ¶µ ¶
1 1
+ −1 −1 .
a b

Second Solution: (by Ian Le) We again prove the stronger inequality (1), which can be rewritten
X
[a − 2(ab)1/2 + (abc)1/3 ] ≥ 0,
sym

where the sum is taken over all six permutations of a, b, c. This inequality follows from adding the
two inequalities X
[a − 2a2/3 b1/3 + (abc)1/3 ] ≥ 0
sym
and X
[a2/3 b1/3 + a1/3 b2/3 − 2a1/2 b1/2 ] ≥ 0.
sym
Mathematics Olympiad Coachs Seminar, Zhuhai, China 11

The first of these is the Schur’s inequality with x = a1/3 , y = b1/3 , z = c1/3 , while the second
follows from the AM-GM inequality.

Third Solution: Without loss of generality, assume that b is between a and c. The desired
inequality reads √ √
3
a + b + c − 3 abc ≤ 3(c + a − 2 ac).
As a function of b, the right side minus the left side is concave (its second derivative is −(2/3)(ac)1/3 b−5/3 ),
so its minimum value in the range [a, c] occurs at one of the endpoints. Thus, without loss of gener-
ality, we may assume a = b. Moreover, we may rescale the variables to get a = b = 1. Now the claim
reads
2c + 3c1/3 + 1
≥ c1/2 .
6
This is an instance of weighted AM-GM inequality.

Note: More generally, for nonnegative real numbers a1 , a2 , . . . , an , we have


m a1 + a2 + · · · + an √ (n − 1)M
≤ − n a1 a2 · · · an ≤ , (2)
2 n 2
where
√ √ √ √
m= min {( ai − aj )2 } and M = max {( ai − aj )2 }.
1≤i<j≤n 1≤i<j≤n

The right inequality can be proved, by using the method of the third solution above. We leave the
details as an exercise for the reader.
√ √
The left inequality falls apart when we replace m by c, the average of ( ai − aj )2 for 1 ≤ i < j ≤ n.
Since
P √ √ 2
m c 1≤i<j≤n ( ai − aj )
≤ = ¡ ¢
2 2 2 n2
P √ √ 2
1≤i<j≤n ( ai − aj )
=
n(n − 1)
P √
(n − 1)(a1 + a2 + · · · + an ) − 2 1≤i<j≤n ai aj
= ,
n(n − 1)
the inequality now reads √
X √ n(n − 1) n a1 a2 · · · an
ai aj ≥ .
2
1≤i<j≤n

This follows from the AM-GM inequality.


We may also replace m by
√ √
m0 = min {( ak − ak+1 )2 }
1≤k≤n

in (2) to obtain, in a way, a sharper lower bound. A similar proof works. We leave it to the reader
as an exercise.
Even more generally, one can ask for a comparison between the difference between the arithmetic and
geometric means of a set of n nonnegative real numbers, and the maximum (or average) difference
between the arithmetic and geometric means over all k-element subsets. The authors do not know
what the correct inequalities should look like or how they may be proved.
12 Zuming Feng (zfeng@exeter.edu), Phillips Exeter Academy, Exeter 03833, USA

11. Let n be a positive integer. Prove that

n µ ¶−1
X n+1
n n + 1 X 2i
= .
i 2n+1 i
i=0 i=1

Solution: Let
n µ ¶ n
1 X n −1 X k!(n − k)!
Sn = = .
n+1 k (n + 1)!
k=0 k=0
P
We must show that Sn = ( n+1 k
k=1 2 /k)/2
n+1 . To do so, it suffices to check that S = 1, which is
1
clear, and that 2n+2 Sn+1 − 2n+1 Sn = 2n+2 /(n + 2). Now
 
n+1 µ ¶ n+1 µ ¶
1 X n + 1 −1 X n + 1 −1 
2Sn+1 = +
n+2 i j
i=0 j=0

n
õ ¶ µ ¶ !
2 1 X n + 1 −1 n + 1 −1
= + +
n+2 n+2 i i+1
i=0
n
2 1 X i!(n + 1 − i)! + (i + 1)!(n − i)!
= +
n+2 n+2 (n + 1)!
i=0
n
2 1 X i!(n − i)!(n + 1 − i + i + 1)
= +
n+2 n+2 (n + 1)!
i=0

2
= Sn + ,
n+2

as claimed.

12. Express
n
X
(−1)k (n−k)!(n+k)!
k=0

in closed form.

Solution: (By Tiankai Liu) Let

f (k) = (n+1−k)!(n+k)!

for integers 0 ≤ k ≤ n + 1. Note that

f (k) + f (k + 1)
= (n+1−k)!(n+k)! + (n−k)!(n+k+1)!
= (n + 1 − k + n + k + 1)(n−k)!(n+k)!
= 2(n + 1)(n−k)!(n+k)!.
Mathematics Olympiad Coachs Seminar, Zhuhai, China 13

Therefore,
n
X
(−1)k (n−k)!(n+k)!
k=0
X n
1
= (−1)k [f (k) + f (k + 1)]
2(n + 1)
k=0
f (0) + (−1)n f (n + 1)
=
2(n + 1)
(n+1)!n! + (−1)n 0!(2n+1)!
=
2(n + 1)
(n!)2 (−1)n (2n+1)!
= + .
2 2(n + 1)

13. Prove that, for all positive real numbers a, b, c,

(a3 + b3 + abc)−1 + (b3 + c3 + abc)−1 + (c3 + a3 + abc)−1 ≤ (abc)−1 .

Solution: The inequality (a − b)(a2 − b2 ) ≥ 0 implies a3 + b3 ≥ ab(a + b), so


1 1 c
≤ = .
a3 3
+ b + abc ab(a + b) + abc abc(a + b + c)

Similarly
1 1 a
≤ = ,
b3 + c3 + abc bc(b + c) + abc abc(a + b + c)
and
1 1 b
≤ = .
c3 3
+ a + abc ca(c + a) + abc abc(a + b + c)
Thus
1 1 1 a+b+c 1
+ 3 + 3 ≤ = .
a3 3 3 3
+ b + abc b + c + abc c + a + abc abc(a + b + c) abc

14. Let a, b, c be positive real numbers such that

a + b + c ≥ abc.

Prove that at least two of the inequalities


2 3 6 2 3 6 2 3 6
+ + ≥ 6, + + ≥ 6, + + ≥6
a b c b c a c a b
are true.

Solution: (by David Shin) Perform the substitutions x = 1/a, y = 1/b, and z = 1/c. It suf-
fices to prove that at least two of the inequalities

2x + 3y + 6z ≤ 6, 2y + 3z + 6x ≤ 6, 2z + 3x + 6y ≤ 6
14 Zuming Feng (zfeng@exeter.edu), Phillips Exeter Academy, Exeter 03833, USA

are true, where


x, y, z > 0 and xy + yz + zx ≥ 1.
Assume, for the sake of contradiction, that at least two of the given inequalities are false. Without
loss of generality, we may assume that 2x + 3y + 6z < 6 and 2y + 3z + 6x < 6. Then

144 > [(2x + 3y + 6z) + (2y + 3z + 6x)]2


= (8x + 5y + 9z)2
= 64x2 + 80xy + 25y 2 + 81z 2 + 90yz + 144zx
= 64x2 − 64xy + 16y 2 + 9y 2 − 54yz + 81z 2
+144(xy + yz + zx)
= (8x − 4y)2 + (3y − 9z)2 + 144 ≥ 144,

a contradiction. Thus, our assumptions is false and at least two of the desired inequalities must be
true. To obtain equalities, that is two of the numbers
2 3 6 2 3 6 2 3 6
+ + , + + , + +
a b c b c a c a b
are equal to 6, we must have 8x − 4y = 3y − 9z = 0, or a : b : c = (1/x) : (1/y) : (1/z) = 2 : 1 : 3.
Therefore the equalities hold if and only if (a, b, c) = (1, 3, 2), (3, 2, 1), (2, 1, 3).

15. Prove that


(a2 + 2)(b2 + 2)(c2 + 2) ≥ 9(ab + bc + ca)
for all positive real numbers a, b, and c.

16. Let a0 , a1 , · · · , an be numbers from the interval (0, π/2) such that
π π π
tan(a0 − ) + tan(a1 − ) + · · · + tan(an − ) ≥ n − 1.
4 4 4
Prove that
tan a0 tan a1 · · · tan an ≥ nn+1 .

Solution: Let bk = tan(ak − π/4), k = 0, 1, . . . , n. It follows from the hypothesis that for each k,
−1 < bk < 1, and X
1 + bk ≥ (1 − bl ). (1)
0≤l6=k≤n

Applying AM-GM to the positive numbers 1 − bl , l = 0, 1, . . . , k − 1, k + 1, . . . , n, we obtain


 1/n
X Y
(1 − bl ) ≥ n  (1 − bl ) . (2)
0≤l6=k≤n 0≤l6=k≤n

From (1) and (2) it follows that

n
à n
!1/n
Y Y
(1 + bk ) ≥ nn+1 (1 − bl )n ,
k=0 l=0
Mathematics Olympiad Coachs Seminar, Zhuhai, China 15

and hence that


n
Y 1 + bk
≥ nn+1 .
1 − bk
k=0

Because ³³
1 + bk 1 + tan(ak − π4 ) π´ π´
= = tan ak − + = tan ak ,
1 − bk 1 − tan(ak − π4 ) 4 4
the conclusion follows.

17. Let R be the set of real numbers. Determine all functions f : R → R such that

f (x2 − y 2 ) = xf (x) − yf (y)

for all pairs of real numbers x and y.

Solution: Setting x = y = 0 in the given condition yields f (0) = 0. Since

−xf (−x) − yf (y) = f ([−x]2 − y 2 ) = f (x2 − y 2 )


= xf (x) − yf (y),

we have f (−x) = −f (x) for x 6= 0. Hence f (x) is odd. From now on, we assume x, y ≥ 0.
Setting y = 0 in the given condition yields f (x2 ) = xf (x). Hence f (x2 − y 2 ) = f (x2 ) − f (y 2 ), or,
f (x2 ) = f (x2 − y 2 ) + f (y 2 ). Since for x ≥ 0 there is a unique t ≥ 0 such that t2 = x, it follows that

f (x) = f (x − y) + f (y) (1)

Setting x = 2t and y = t in (1) gives


f (2t) = 2f (t). (2)
Setting x = t + 1 and y = t in the given condition yields

f (2t + 1) = (t + 1)f (t + 1) − tf (t). (3)

By (2) and by setting x = 2t + 1 and y = 1 in (1), the left-hand side of (3) becomes

f (2t + 1) = f (2t) + f (1) = 2f (t) + f (1). (4)

On the other hand, by setting x = t + 1 and y = 1 in (1), the right-hand side of (3) reads

(t + 1)f (t + 1) − tf (t) = (t + 1)[f (t) + f (1)] − tf (t),

or,
(t + 1)f (t + 1) − tf (t) = f (t) + (t + 1)f (1). (5)
Putting (3), (4), and (5) together leads to 2f (t) + f (1) = f (t) + (t + 1)f (1), or,

f (t) = tf (1)

for t ≥ 0. Recall that f (x) is odd; we conclude that f (−t) = −f (t) = −tf (1) for t ≥ 0. Hence
f (x) = kx for all x, where k = f (1) is a constant. It is not difficult to see that all such functions
indeed satisfy the conditions of the problem.
16 Zuming Feng (zfeng@exeter.edu), Phillips Exeter Academy, Exeter 03833, USA

18. Let a, b, and c be nonnegative real numbers such that

a2 + b2 + c2 + abc = 4.

Prove that
0 ≤ ab + bc + ca − abc ≤ 2.

First Solution: (By Richard Stong) From the condition, at least one of a, b, and c does not
exceed 1, say a ≤ 1. Then

ab + bc + ca − abc = a(b + c) + bc(1 − a) ≥ 0.

To obtain equality, we have a(b + c) = bc(1 − a) = 0. If a = 1, then b + c = 0 or b = c = 0, which


contradicts the given condition a2 + b2 + c2 + abc = 4. Hence 1 − a 6= 0 and only one of b and c is 0.
Without loss of generality, say b = 0. Therefore b + c > 0 and a = 0. Plugging a = b = 0 back into
the given condition gives c = 2. By permutation, the lower bound holds if and only if (a, b, c) is one
of the triples (2, 0, 0), (0, 2, 0), and (0, 0, 2).
Now we prove the upper bound. Let us note that at least two of the three numbers a, b, and c are
both greater than or equal to 1 or less than or equal to 1. Without loss of generality, we assume that
the numbers with this property are b and c. Then we have

(1 − b)(1 − c) ≥ 0. (1)

The given equality a2 + b2 + c2 + abc = 4 and the inequality b2 + c2 ≥ 2bc imply

a2 + 2bc + abc ≤ 4, or bc(2 + a) ≤ 4 − a2 .

Dividing both sides of the last inequality by 2 + a yields

bc ≤ 2 − a. (2)

Combining (1) and (2) gives

ab + bc + ac − abc ≤ ab + 2 − a + ac(1 − b)
= 2 − a(1 + bc − b − c)
= 2 − a(1 − b)(1 − c) ≤ 2,

as desired.
The last equality holds if and only if b = c and a(1−b)(1−c) √ = 0.√Hence,
√ equality
√ for the√upper
√ bound
holds if and only if (a, b, c) is one of the triples (1, 1, 1), (0, 2, 2), ( 2, 0, 2), and ( 2, 2, 0).

Second Solution: (by Oaz Nir) We prove only the upper bound here. Either two of a, b, c
are less than or equal to 1, or two are greater than or equal to 1. Assume b and c have this property.
Then
b + c − bc = 1 − (1 − b)(1 − c) ≤ 1. (3)
Viewing the given equality as a quadratic equation in a and solving for a yields
p
−bc ± (b2 − 4)(c2 − 4)
a= .
2
Mathematics Olympiad Coachs Seminar, Zhuhai, China 17

Note that

(b2 − 4)(c2 − 4) = b2 c2 − 4(b2 + c2 ) + 16


≤ b2 c2 − 8bc + 16 = (4 − bc)2 .

For the given equality to hold, we must have b, c ≤ 2 so that 4 − bc ≥ 0. Hence,


−bc + |4 − bc| −bc + 4 − bc
a≤ = = 2 − bc,
2 2
or
2 − bc ≥ a. (4)
Combining (3) and (4) gives

2 − bc ≥ a(b + c − bc) = ab + ac − abc,

or
ab + ac + bc − abc ≤ 2,
as desired.

Third Solution: We prove only the upper bound here. Define functions f , g as

f (x, y, z) = x2 + y 2 + z 2 + xyz = (x + y)2 + z 2 − (2 − z)xy,


g(x, y, z) = xy + yz + zx − xyz = z(x + y) + (1 − z)xy

for all nonnegative numbers x, y, z. Observe that if z ≤ 1, then both f and g are unbounded,
increasing functions of x and y.
Assume that f (a, b, c) = 4 and, without loss of generality, that a ≥ b ≥ c ≥ 0. Then c ≤ 1.
¡ ¢2
Let a0 = (a + b)/2. Because a + b = a0 + a0 and ab ≤ a−b 2 + ab = a0 2 , we have

f (a0 , a0 , c) ≤ f (a, b, c) = 4 and g(a0 , a0 , c) ≥ g(a, b, c).

Now increase a0 to e ≥ 0 such that f (e, e, c) = 4. Note that g(e, e, c) ≥ g(a0 , a0 , c). It suffices to prove
that g(e, e, c) ≤ 2.
Since f (e, e, c) = 2e2 + c2 + e2 c = 4, e2 = (4 − c2 )/(2 + c) = 2 − c. We obtain that

g(e, e, c) = 2ec + (1 − c)e2 ≤ e2 + c2 + (1 − c)e2


= (2 − c)e2 + c2 = (2 − c)2 + c2
= 2(2 − 2c + c2 ) = 2[1 + (1 − c)2 ] ≤ 2,

as desired.

19. Let a, b, and c be positive real numbers, not all equal. Find all solutions to the system of equations

x2 − yz = a,
y 2 − zx = b,
z 2 − xy = c,

in real numbers x, y, and z.


18 Zuming Feng (zfeng@exeter.edu), Phillips Exeter Academy, Exeter 03833, USA

Solution: Squaring each equation and subtracting the product of the other two yields

a2 − bc = x(x3 + y 3 + z 3 − 3xyz),
b2 − ca = y(x3 + y 3 + z 3 − 3xyz),
c2 − ab = z(x3 + y 3 + z 3 − 3xyz).

Let k = x3 + y 3 + z 3 − 3xyz. Then

(a2 − bc)2 − (b2 − ca)(c2 − ab) = k 2 (x2 − yz) = k 2 a.

The same computation that produced the system above shows tha the expression on the left is
a(a3 + b3 + c3 − 3abc), and the latter is positive by the AM-GM inequality. Hence
p
k = ± a3 + b3 + c3 − 3abc

and the solutions to the system (one for each choice of k) are
a2 − bc b2 − ca c2 − ab
x= , y= , and z = .
k k k
20. Find all functions f : N → N satisfying

f (f (f (n))) + 6f (n) = 3f (f (n)) + 4n + 2001,

for all n ∈ N.

Solution: We first look for a function of the form f (n) = n + a. The relation from the state-
ment yields a = 667, and hence f (n) = n + 667 is a solution. Let us show that this is the only
solution.
Fix some positive integer n and define a0 = n, and ak = f (ak−1 0 for k ≥ 1. The sequence {ak }k≥0
satisfies the recursive relation

ak+3 − 3ak+2 + 6ak+1 − 4ak = 2001 (1)

A particular solution is ak = 667k. The characteristic equation of the homogeneous recurrence


ak+3 − 3ak+2 + 6ak+1 − 4ak = 0 is

x3 − 3x2 + 6x − 4 = 0.

It is not hard to guess that x1 = 1 is a solution to this equation. Since x3 − 3x2 + 6x + 4 =


(x − 1)(x2 − 2x + 4), the other two zeros are x2 = 2 cis 120◦ and x3 = 2 cis (−120◦ ). It follows that
the general solution to (1) is of the form

ak = c1 + 2k c2 cos(120◦ n) + 2k c3 sin(120◦ n) + 667k

with c1 , c2 , c3 some real constants.


if c2 > 0, then a3(2m+1) will be negative for large m and if c2 < 0, then a6m will be negative for large
m. Since f can only take positive values, this implies that c2 = 0. A similar argument shows that
c3 = 0. It follows that ak = c1 + 667k. So the first term of the sequence determine all the others.
Since a0 = n, c1 = n, and hence ak = n + 667k, for all k. In particular a1 = f (n) = n + 667, and
hence this is the only possible solution.
Mathematics Olympiad Coachs Seminar, Zhuhai, China 19

21. Let a1 , . . . , an and b1 , . . . , bn be two sequences of distinct numbers such that ai + bj 6= 0 for all i, j.
Show that if ½
Xn
cj,k 1 if i = k
=
ai + bj 0 otherwise,
j=1

then the sum of all n2 numbers cjk is (a1 + · · · + an ) + (b1 + · · · + bn ).

Pn
Solution: Let rj = k=1 cj,k . Then
n
X n Pn
X n X
X n n X
X n
rj k=1 cj,k cj,k cj,k
= = = = 1, (1)
ai + bj ai + bj ai + bj ai + bj
j=1 j=1 j=1 k=1 k=1 j=1

for all i = 1, 2, . . . , n. We wish to determine r1 + r2 + · · · + rn . Let


n
X rj
R(x) = . (2)
x + bj
j=1

Then R(x) = P (x)/Q(x) where Q(x) = (x + b1 )(x + b2 ) · · · (x + bn ) and P (x) has degree at most
n − 1. By (1), R(a1 ) = R(a2 ) = · · · = R(an ) = 1, so if we write

S(x)
R(x) = 1 − ,
Q(x)

then S(x) is a monic polynomial of degree n and S(a1 ) = S(a2 ) = · · · = S(an ) = 0. Hence

S(x) = (x − a1 )(x − a2 ) · · · (x − an ).

Consider the coefficient of xn−1 in P (x) = Q(x) − S(x). Form (2), this coefficient is r1 + r2 + · · · + rn .
On the other hand,

Q(x) = (x + b1 )(x + b2 ) · · · (x + bn ) and S(x) = (x − a1 )(x − a2 ) · · · (x − an ).

Applying the Vieta’s theorem, this coefficient is (a1 + a2 + · · · + an ) + (b1 + b2 + · · · + bn ). Hence


we have our desired result.

22. Suppose the positive integers have been expressed as a disjoint union of arithmetic progressions
{ai + ndi }∞
n=0 , i = 1, 2, . . . , k. Show that

k
X ai k+1
= .
di 2
i=1

Solution: Note that


k
X xai x
d
= ,
1−x i 1−x
i=1
or
k
X xai −1 1
d
= , (1)
1−x i 1−x
i=1
20 Zuming Feng (zfeng@exeter.edu), Phillips Exeter Academy, Exeter 03833, USA

as each power of x appears with coefficient 1 on both sides of the equations. Multiplying both sides
of (1) by (1 − x) yields
X k
xai −1
= 1.
1 + x + · · · + xdi −1
i=1
Setting x = 1 in the last equation gives
k
X 1
= 1.
di
i=1
We now rewriting (1) as
k µ ai −1
X ¶ k
X 1
x −1 1 di
+ = ,
1 − xdi 1 − xdi 1−x
i=1 i=1
that is, Ã !
k
X 1 k
X
1 di 1 + x + · · · + xai −2
− = .
1 − xdi 1−x 1 + x + · · · + xdi −1
i=1 i=1
Hence
k
X k
X
di − 1 − x − · · · − xdi −1 1 + x + · · · + xai −2
= ,
di (1 − xdi ) 1 + x + · · · + xdi −1
i=1 i=1
or
k
X k
X
(1 − x)[(di − 1) + (di − 2)x + · · · + xdi −1 ] 1 + x + · · · + xai −2
= ,
di (1 − x)(1 + x + · · · + xdi −1 ) 1 + x + · · · + xdi −1
i=1 i=1
Setting x = 1 in the last equation yields
k
X k
X
(di − 1) + (di − 2) + · · · + 1 ai − 1
= ,
i=1
d2i i=1
di
or
k
X k
X k
X
di (di − 1) ai 1
= − .
i=1
2d2i i=1
di di
i=1
Hence
k
X k µ
X ¶
ai 1 1 k+1
= + = ,
di 2 2di 2
i=1 i=1
as desired.
23. Let b
P S be the set of perfect powers, that is, the numbers of the form a for some a, b > 1. Show that
n∈S 1/(n − 1) = 1.

Solution: Note that


∞ µ
∞ X ¶ X∞
à ! ∞
X 1 1 1 X 1
= · 1 = = 1.
ab 2
a 1− a
a(a − 1)
a=2 b=2 a=2 a=2

On the other hand, the desired sum


X ∞
XX
1 1
= .
n−1 nm
n∈S n∈S m=1
Mathematics Olympiad Coachs Seminar, Zhuhai, China 21

But this is just the same as my first sum. Indeed, for any denominator d, the number of times 1/d
occurs in the first sum is the number of representations d = ab (a, b ≥ 2), while the number of times
1/d occurs in the second sum is the number of representations d = nm (n ∈ S, m ≥ 1). If we write
d = pq with p minimal, then both these multiplicities are equal to f (q) − 1, where f (q) denote the
number of divisors of q. So the sums really are the same.

24. Prove that the product of any k consecutive Fibonacci numbers is divisible by the product of the
first k Fibonacci numbers.

Solution: Let [k]! = F1 F2 · · · Fk for k ≥ 1 and [0]! = 1, and let Ra,b = [a + n]!/([a]![b]!)
√ for
a, b ≥ 0. √We need to show that Ra,b is an integer for all a, b ≥ 0. Putting α = (1 + 5)/2 and
β = (1 − 5)/2, we have

αn − β n
Fn = = αn−1 + αn−2 β + · · · + β n−1 (1)
α−β

for all n ≥ 1. From this one can derive the recursice relation

Ra,b = αa Ra,b−1 + β b Rx−1,b (2)

for a, b ≥ 1, since dividing both sides of (2) by [a + b − 1]!/([a]![b]!) converts it into the formula

Fa+b = αa Fb + β b Fa

which in turn follows easily from (1). The recursive relation (2), combined with the initial conditions
Ra,0 = R0,b = 1 for all a, b ≥ 0, guarantees that Ra,b is always expressible as a polynomial in α and
β with integer coefficients.
Now since β = 1 − α, Ra,b can be expressed as a polynomial in α with integer coefficients. Conse-
quently, iteratively applying αn = αn−1 + αn−2 for n ≥ 2, we can write Ra,b in the form r + sα, where
r and s are integers. Note that Ra,b is rational and α is irrational. Hence s = 0 and Ra,b = r is an
integer.

25. Prove that any monic polynomial (a polynomial with leading coefficient 1) of degree n with real
coefficients is the average of two monic polynomials of degree n with n real roots.

First Solution: (Tiankai Liu) Let us begin with the following lemma.
Lemma. For any set of n ordered pairs of real numbers {(xi , yi )}ni=1 , with xi 6= xj for all i 6= j,
there exists a unique monic real polynomial P (x) of degree n such that P (xi ) = yi for all integers
i ∈ [1, n].
We present two proofs of the lemma.

• First proof By the Lagrange Interpolation Formula, there exists a unique real polynomial
Q(x) of degree less than n such that Q(xi ) = yi − xni for all integers i = 1, 2, . . . , n. Then we
can, and must, have P (x) = Q(x) + xn .
• Second proof By the Lagrange Interpolation Formula, there exists a unique real polynomial
Q(x) of degree less than n suchQthat Q(xi ) = yi for all integers i = 1, 2, . . . , n. Then we can,
and must, have P (x) = Q(x) + ni=1 (x − xi ).
22 Zuming Feng (zfeng@exeter.edu), Phillips Exeter Academy, Exeter 03833, USA

Let the given monic real polynomial be F (x). Choose any strictly decreasing sequence of n real
numbers x1 , . . . , xn . For each odd i, choose a yi such that yi < min{0, 2F (xi )}. For each even i,
choose a yi such that yi > max{0, 2F (xi )}.
Let P (x) be the monic real polynomial of degree n such that P (xi ) = yi for all integers i = 1, 2, . . . , n;
the existence of such a P (x) is guaranteed by the lemma. Let Q(x) = 2F (x) − P (x).
Note that, for each integer i = 1, 2, . . . , n − 1, P (x) has a root in (xi+1 , xi ) because P (xi+1 ) and
P (xi ) have opposite signs and polynomial functions are continuous. Similarly, for each integer i =
1, 2, . . . , n − 1, Q(X) has a root in (xi+1 , xi ) because 2F (xi+1 ) − P (xi+1 ) and 2F (xi ) − P (xi ) have
opposite signs.
Therefore, we are guaranteed that P (x) and Q(x) each have n roots with at least n − 1 of them are
real. Because imaginary roots of real polynomials come in conjugate pairs, all real polynomials have
an even number of imaginary roots. Hence the other root of P (x) must also be real, and the same
holds for Q(x). Thus, P (x) and Q(x) each have n real roots.

Note: One could also prove that P (x) and Q(x) both have n real roots by the following argu-
ment. Because P (x) has a positive leading coefficient, limx→∞ P (x) = ∞. Because P (x1 ) < 0, then,
P (x) has another root in (x1 , +∞). Similarly, Q(x) also has a leading coefficient of +1. If n is odd,
then limx→−∞ Q(x) = −∞, so because Q(xn ) > 0, Q(x) has another root in (−∞, xn ). If n is even,
then limx→−∞ Q(x) = ∞, so because Q(xn ) < 0, Q(x) has another root in (−∞, xn ).

Second Solution: Let F (x) be the monic real polynomial of degree n. If n = 1, then F (x) = x + a
for some real number a. Then F (x) is the average of x + 2a and x, each of which has 1 real root.
Now we assume that n > 1. Define the polynomial
G(x) = (x − 2)(x − 4) · · · (x − 2(n − 1)).
The degree of G(x) is n − 1. Consider the polynomials
P (x) = xn − kG(x)
and
Q(x) = 2F (x) − P (x) = 2F (x) − xn + kG(x).
We will show that for large enough k these two polynomials have n real roots. Since they are monic
and their average is F (x), this will solve the problem.
Consider the values of polynomial G(x) at n points x = 1, 3, 5, . . . , 2n − 1. These values alternate
in sign and have magnitude at least 1 (because at most two of the factors have magnitude 1 and
the others have magnitude at least 2). On the other hand, there is a constant c > 0 such that for
0 ≤ x ≤ 2n − 1, we have |xn | < c and |2F (x) − xn | < c. Take k > c. Then we see that P (x) and Q(x)
evaluated at n points x = 1, 3, 5, . . . , 2n − 1 alternate in sign. Thus, polynomials P (x) and Q(x) each
have at least n − 1 real roots — one in each interval (1, 3), . . ., (2n − 3, 2n − 1). However, since they
are polynomials of degree n, they must then each have n real roots (as in the previous solution), as
desired.
26. Prove that for any integer n, there exists a unique polynomial Q with coefficients in {0, 1, . . . , 9} such
that Q(−2) = Q(−5) = n.

Solution: First suppose there exists a polynomial Q with coefficients in {0, 1, . . . , 9} such that
Q(−2) = Q(−5) = n. We shall prove that this polynomial is unique. By the Factor Theorem, we can
Mathematics Olympiad Coachs Seminar, Zhuhai, China 23

write Q(x) = P (x)R(x)+n where P (x) = (x+2)(x+5) = x2 +7x+10 and R(x) = r0 +r1 x+r2 x2 +· · ·
is a polynomial. Then r0 , r1 , r2 , . . . are integers such that

10r0 + n, 10rk + 7rk−1 + rk−2 ∈ {0, 1, . . . , 9}, k≥1 (∗)

(with the understanding that r−1 = 0). For each k, (∗) uniquely determines rk once rj is known for
all j < k. Uniqueness of R, and therefore of Q, follows.
Existence will follow from the fact that for the unique sequence {rk } satisfying (∗), there exists some
N such that rk = 0 for all k ≥ N . First note that {rk } is bounded, since |r0 |, |r1 | ≤ B and B ≥ 9
imply |rk | ≤ B for all k. This follows by induction, using 10|rk | ≤ 7|rk−1 | + |rk−2 | + 9 ≤ 10B. More
specifically, if ri ≤ M for i = k − 1, k − 2, then
7rk−1 rk−2 4M
rk ≥ − − ≥− ,
10 10 5
while if ri ≥ L for i = k − 1, k − 2, then
7rk−1 rk−2 9 4L 9
rk ≤ − − + ≤− + .
10 10 10 10 10
Since the sequence {rk } is bounded, we can define

Lk = min{rk , rk−1 , . . . }, Mk = max{rk , rk+1 , . . . }.

Clearly Lk ≤ Lk+1 and Mk ≥ Mk+1 for all k.


Since Lk ≤ Mk for all k, the non-decreasing sequence {Lk } must stop increasing eventually, and, sim-
ilarly, the non-increasing sequence {Mk } must stop decreasing. In other words, there exist L, M, N
such that Lk = L and Mk = M for all k ≥ N . Certainly L ≤ M , and M ≥ 0, since no three
consecutive terms in {rk } can be negative, but the above arguments also imply L ≥ −4M/5 and
M ≤ −4L/5+9/10. A quick sketch shows that the set of real pairs (L, M ) satisfying these conditions
is a closed triangular region containing no lattice points other than (0, 0). It follows that rk = 0 for
all k ≥ N , proving existence.

27. Let a1 , b1 , a2 , b2 , . . . , an , bn be nonnegative real numbers. Prove that


n
X n
X
min{ai aj , bi bj } ≤ min{ai bj , aj bi }.
i,j=1 i,j=1

Solution: (Based on work by George Lee) Define


X
L(a1 , b1 , . . . , an , bn ) = (min{ai bj , aj bi } − min{ai aj , bi bj }).
i,j

Our goal is to show that


L(a1 , b1 , . . . , an , bn ) ≥ 0
for a1 , b1 , . . . , an , bn ≥ 0. Our proof is by induction on n, the case n = 1 being evident. Using the
obvious identities

• L(a1 , 0, a2 , b2 , . . . ) = L(0, b1 , a2 , b2 , . . . ) = L(a2 , b2 , . . . ),


• L(x, x, a2 , b2 , . . . ) = L(a2 , b2 , . . . ),
24 Zuming Feng (zfeng@exeter.edu), Phillips Exeter Academy, Exeter 03833, USA

and the less obvious but easily verified identities

• L(a1 , b1 , a2 , b2 , a3 , b3 , . . . ) = L(a1 + a2 , b1 + b2 , a3 , b3 , . . . ) if a1 /b1 = a2 /b2 ,


• L(a1 , b1 , a2 , b2 , a3 , b3 , . . . ) = L(a2 − b1 , b2 − a1 , a3 , b3 , . . . ) if a1 /b1 = b2 /a2 and a1 ≤ b2 ,

we may deduce the result from the induction hypothesis unless we are in the following situation:

(a) all of the ai and bi are nonzero;


(b) for i = 1, . . . , n, ai 6= bi ;
(c) for i 6= j, ai /bi 6= aj /bj and ai /bi 6= bj /aj .

For i = 1, . . . , n, let ri = max{ai /bi , bi /ai }. Without loss of generality, we may assume 1 < r1 <
· · · < rn , and that a1 < b1 . Now notice that f (x) = L(a1 , x, a2 , b2 , . . . , an , bn ) is a linear function of
x in the interval [a1 , r2 a1 ]. Explicitly,

f (x) = min{a1 x, xa1 } − min{a21 , x2 } + L(a2 , b2 , . . . , an , bn )


Xn
+2 (min{a1 bj , xaj } − min{a1 aj , xbj })
j=2
n
X
= (x − a1 )(a1 + 2 cj ) + L(a2 , b2 , . . . , an , bn ),
j=2

where cj = −bj if aj > bj and cj = aj if aj < bj .


In particular, since f is linear, we have

f (x) ≥ min{f (a1 ), f (r2 a1 )}.

Note that f (a1 ) = L(a1 , a1 , a2 , b2 , . . . ) = L(a2 , b2 , . . . ) and

f (r2 a1 ) = L(a1 , r2 a1 , a2 , b2 , . . . )
½
L(a1 + a2 , r2 a1 + b2 , a3 , b3 , . . . ) if r2 = b2 /a2 ,
=
L(a2 − r2 a1 , b2 − a1 , a3 , b3 , . . . ) if r2 = a2 /b2 .

Thus we deduce the desired inequality from the induction hypothesis in all cases.

Note: More precisely, it can be shown that for ai , bi > 0, equality holds if and only if, for each
r > 1, the set Sr of indices i in {1, . . . , n} such that ai /bi ∈ {r, 1/r} has the property that
X X
ai = bi .
i∈Sr i∈Sr

Namely, assume this is the case for n − 1 pairs. Given n pairs, if conditions (a)-(c) are not all met, we
may deduce the result from the induction hypothesis by the same reductions as that at the beginning
of the proof. If (a)-(c) are met, then for equality to hold, 0 = f (b1 ) ≥ min{f (a1 ), f (r2 a1 )} ≥ 0.
Since f (x) is linear on the interval [a1 , r2 a1 ], f (x) is identically zero on the interval. Since f (a1 ) = 0,
L(a2 , b2 , . . . ) = 0. Applying the induction hypothesis, with all of ai and bi nonzero ((a) is met) and
ri > 1 ((b) is met), we have

(i) for each i ≥ 2, there exists an r > 1 such that either ai = rbi or bi = rai .
Mathematics Olympiad Coachs Seminar, Zhuhai, China 25

n
X
(ii) (aj − bj ) = 0.
j=2

Therefore, if aj > bj , aj = rbj and cjP (1 − r) = (−bj )(1 − r) = aj − bj ; if aj < bj , raj = bj and
cj (1 − r) = aj (1 − r) = aj − bj . Hence nj=2 cj = 0. But then 0 = f (r2 a1 ) yields
n
X
0 = (r2 a1 − a1 )(a1 + 2 cj ) + L(a2 , b2 , . . . , an , bn ) = (r2 − 1)a21
j=2

and a1 = 0, a contradiction.

You might also like